Matrix Multiplication and Inverses

Click For Summary
The discussion centers on proving that matrix A and the inverse of (I+A) commute, with the assumption that the inverse of A may not exist. The solution provided attempts to demonstrate the relationship without assuming A's invertibility, focusing instead on the properties of the identity matrix and the existence of the inverse of (I+A). Participants clarify that if (I+A) has an inverse, it suffices to show that A and Inverse(I+A) commute without needing to prove A's inverse exists. The conclusion emphasizes that the proof can be valid even if A is not invertible, as demonstrated by the example of A being the zero matrix. The discussion ultimately confirms that the commuting property holds under the given conditions.
Random Variable
Messages
114
Reaction score
0

Homework Statement



Show that A and Inverse(I+A) commute (where I is the identity matrix).


Homework Equations



Inverse(Inverse(A))=A

Inverse(AB)=Inverse(B)*Inverse(A)


The Attempt at a Solution



My solution assumes the existence of the inverse of A.

A*Inverse(I+A) = Inverse(Inverse(A))*Inverse(I+A)
= Inverse[(I+A)*Inverse(A)]
= Inverse[Inverse(A)+I]
= Inverse[Inverse(A)*(I+A)]
= Inverse(I+A)*Inverse(Inverse(A))
= Inverse(I+A)*A

My professor told me that the inverse of A may or may not exist. Does he want me to prove that it does exist? Can you even prove it does exist from the fact that the inverse of (I+A) exists? Does he want a different proof? Is he just giving me a hard time?
 
Physics news on Phys.org
You can't prove A^(-1) exists. It might not. Suppose A=0. The result is still true. If (I+A)^(-1) exists then you must have (I+A)*(I+A)^(-1)=(I+A)^(-1)*(I+A)=I. Isn't that enough to prove it without assuming A^(-1) exists?
 
Yes, that's all you need. Thanks.
 
Question: A clock's minute hand has length 4 and its hour hand has length 3. What is the distance between the tips at the moment when it is increasing most rapidly?(Putnam Exam Question) Answer: Making assumption that both the hands moves at constant angular velocities, the answer is ## \sqrt{7} .## But don't you think this assumption is somewhat doubtful and wrong?

Similar threads

  • · Replies 10 ·
Replies
10
Views
2K
  • · Replies 12 ·
Replies
12
Views
3K
  • · Replies 6 ·
Replies
6
Views
2K
  • · Replies 20 ·
Replies
20
Views
2K
  • · Replies 8 ·
Replies
8
Views
3K
  • · Replies 4 ·
Replies
4
Views
2K
  • · Replies 7 ·
Replies
7
Views
2K
Replies
9
Views
2K
  • · Replies 2 ·
Replies
2
Views
2K
Replies
4
Views
5K